LSAT and Law School Admissions Forum

Get expert LSAT preparation and law school admissions advice from PowerScore Test Preparation.

 Administrator
PowerScore Staff
  • PowerScore Staff
  • Posts: 8917
  • Joined: Feb 02, 2011
|
#92256
Complete Question Explanation

The correct answer choice is (B).

Answer choice (A):

Answer choice (B): This is the correct answer choice.

Answer choice (C):

Answer choice (D):

Answer choice (E):

This explanation is still in progress. Please post any questions below!
 kcho10
  • Posts: 69
  • Joined: Nov 02, 2015
|
#33092
I understand why B is correct. However, I had a very tough time eliminating E.

I get that paragraph 2 says that the book 'received a mixture of positive and negative reviews' (18-19) which does not suggest that MOST reviewers responded a certain way. But it later says 'MOST critics' and readers' expectations of black literature rendered them unable to appreciate Hurston's subtle delineation...and the novel went quietly out of print". How is E incorrect? Is it because they may have not appreciated this aspect of Their Eyes, but still had an overall positive response to the book?
User avatar
 Jonathan Evans
PowerScore Staff
  • PowerScore Staff
  • Posts: 726
  • Joined: Jun 09, 2016
|
#33129
Hi, Kcho,

Excellent question and analysis of this question, text, and answer choices.

In fact, your explanation is largely correct. Since we have to find an answer choice that has explicit support in the passage, answer choice (E) actually fails this criterion since it is possible that on balance the reviewers were still able to respond positively to the text even though they were ill-equipped to appreciate the "subtle delineation of the life of an ordinary Black woman in a Black community."

However, there is an even stronger case against (E) than this observation: The statement in line 36 addresses the responses of both critics and readers. Answer choice (E) contends that a majority of early reviewers of the text responded negatively. When you lump critics together with readers, it becomes impossible to make an inference about only the critics; therefore this answer choice is not supported.

Great question and observations!

Get the most out of your LSAT Prep Plus subscription.

Analyze and track your performance with our Testing and Analytics Package.